MCAT General Chemistry Practice Test 6: Equilibrium

Home > MCAT Test > MCAT general chemistry practice tests

Test Information

Question 15 questions

Time minutes

See All test questions

Take more free MCAT general chemistry practice tests available from maintests.com.

1. A reaction is found to stop just before all reactants are converted to products. Which of the following could be true about this reaction?

  • A. The reaction is irreversible, and the forward rate is greater than the reverse rate.
  • B. The reaction is irreversible, and the reverse rate is too large for products to form.
  • C. The reaction is reversible, and the forward rate is equal to the reverse rate.
  • D. The reaction is reversible, and the reverse rate is greater than the forward rate.

2. What is the equilibrium expression for the reaction Cu2SO4 (s) ? 2 Cu+ (aq) + SO42- (aq)?

  • A.
  • B.
  • C.
  • D.

3. Carbonated beverages are produced by dissolving carbon dioxide in water to produce carbonic acid:

CO2 (g) + H2O (l) ? H2CO3 (aq)

When a bottle containing carbonated water is opened, the taste of the beverage gradually changes as the carbonation is lost. Which of the following statements best explains this phenomenon?

  • A. The change in pressure and volume causes the reaction to shift to the left, thereby decreasing the amount of aqueous carbonic acid.
  • B. The change in pressure and volume causes the reaction to shift to the right, thereby decreasing the amount of gaseous carbon dioxide.
  • C. Carbonic acid reacts with environmental oxygen and nitrogen.
  • D. Carbon dioxide reacts with environmental oxygen and nitrogen.

4. What is the proper equilibrium expression for the reaction below?

2 NO2 (g) + 4 H2 (g) ? N2 (g) + 4 H2O (g)

  • A.
  • B.
  • C.
  • D.

5. If Kc ? 1:

  • A. the equilibrium mixture will favor products over reactants.
  • B. the equilibrium mixture will favor reactants over products.
  • C. the equilibrium concentrations of reactants and products are equal.
  • D. the reaction is essentially irreversible.

6. Acetic acid dissociates in solution according to the following equation:

CH3COOH ? CH3COO + H+

If sodium acetate is added to a solution of acetic acid in excess water, which of the following effects would be observed in the solution?

  • A. Decreased pH
  • B. Increased pH
  • C. Decreased pKeq (pKa)
  • D. Increased pKeq (pKa)

7. Given the reaction below:

FeI (aq) + I2 (g) → FeI3 (aq)

Which of the following would increase the formation of product?

  • A. Decreasing the volume of the container
  • B. Decreasing the pressure of the container
  • C. Increasing the volume of the container
  • D. Decreasing the volume of the container while maintaining a constant pressure

8. If the reaction FeI (aq) + I2 (g) → FeI3 (aq) were exothermic, what effect would decreasing the temperature have on the equilibrium?

  • A. The forward reaction rate and the reverse reaction rate both increase.
  • B. The forward reaction rate decreases while the reverse reaction rate increases.
  • C. The forward reaction rate increases while the reverse reaction rate decreases.
  • D. The forward reaction rate and the reverse reaction rate both decrease.

9. Which of the following actions does NOT affect the equilibrium position of a reaction?

  • A. Adding or subtracting heat.
  • B. Adding or removing a catalyst.
  • C. Increasing or decreasing concentrations of reactants.
  • D. Increasing or decreasing volumes of reactants.

10. In a sealed 1 L container, 1 mole of nitrogen gas reacts with 3 moles of hydrogen gas to form 0.05 moles of NH3 at equilibrium. Which of the following is closest to the Kc of the reaction?

  • A. 0.0001
  • B. 0.001
  • C. 0.01
  • D. 0.1

11. Increasing temperature can alter the Keq of a reaction. Why might increasing temperature indefinitely be unfavorable for changing reaction conditions?

  • A. The equilibrium constant has a definite limit that cannot be surpassed.
  • B. The products or reactants can decompose at high temperatures.
  • C. Increasing temperature would decrease pressure, which may or may not alter reaction conditions.
  • D. If a reaction is irreversible, its Keq will resist changes in temperature.

12. Which of the following is true of equilibrium reactions?

I. An increase in k1 results in a decrease in k–1.

II. As the concentration of products increases, the concentrations of reactants decreases.

III. The equilibrium constant is altered by changes in temperature.

  • A. I only
  • B. II and III only
  • C. I and III only
  • D. I, II, and III

13. Compound A has a Ka (equilibrium constant of acid dissociation) of approximately 10–4. Which of the following compounds is most likely to react with a solution of compound A?

  • A. HNO3
  • B. NO2
  • C. NH3
  • D. N2O5

14. Consider the following two reactions:

3 A + 2 B ? 3 C + 4 D (Reaction 1)

4 D + 3 C ? 3 A + 2 B (Reaction 2)

If Keq for reaction 1 is equal to 0.1, what is Keq for reaction 2?

  • A. 0.1
  • B. 1
  • C. 10
  • D. 100

15. Which of the following statements best describes the effect of lowering the temperature of the following reaction?

  • A. [C] and [D] would increase.
  • B. [A] and [B] would increase.
  • C. ΔH would increase.
  • D. ΔH would decrease.